The table shows the​ distribution, by age and​ gender, of the million people in a certain region who live alone. Use the data in the table to find the probability that a randomly selected person in the region is a woman in the 1824 age range living alone.

Answers

Answer 1

The probability that a randomly selected person in the region is a woman in the 18-24 age range living alone is approximately 0.14 or 14%.

To find the probability that a randomly selected person in the region is a woman in the 18-24 age range living alone, we need to look at the intersection of the "Female" row and the "0-24" age column in the table.

The probability of selecting a woman in the 18-24 age range living alone is:

P(woman, 18-24, living alone) = (Number of women in the 18-24 age range living alone) / (Total number of people living alone)

From the table, we can see that the number of women in the 18-24 age range living alone is 20.9. The total number of people living alone is 153.6.

P(woman, 18-24, living alone) = 20.9 / 153.6

P(woman, 18-24, living alone) = 0.1362 or approximately 0.14

To learn more about the probability;

brainly.com/question/11234923

#SPJ1

The complete question:

The table shows the​ distribution, by age and​ gender, of the million people in a certain region who live alone. Use the data in the table to find the probability that a randomly selected person in the region is a woman in the 1824 age range living alone. Age |0-24|25-49|50+ |Total

Male|21.9|25.3 |28.6|75.8

Fem |20.9|26.1 |30.8|77.8

Tot |42.8|51.4 |59.4|153.6


Related Questions

A circular arc of length 9 feet subtends a central angle of 65 degrees. Find the radius
of the circle in feet.

Answers

The radius of the circle is approximately 7.9 feet.

What is the radius of the circle?

The arc length formula (if θ is in degrees) is expressed as:

s = 2πr × (θ/360°)

Where r is radius and θ is the central angle subtended by the arc.

Given that:

Arc length s = 9ft

Central angle subtended by the arc = 65°

Radius r = ?

Plug the given values into the above formula.

s = 2πr × (θ/360°)

9ft = 2 × 3.14 × r × (65/360°)

r = 9ft / 1.133888

r = 7.9 ft

Therefore, the radius is 7.9 ft.

Learn more about arc of length here: https://brainly.com/question/16403495

#SPJ1

1. Out of 318 seventh and eighth grade CAVA students there were 48 students that chose Music as their elective
course and the rest chose World Language. There were a total of 124 eighth grade students and 108 of them
chose World Language as their elective.
Use this information to complete the two-way table completely.
Enter your answer by filling in the boxes to complete the table (5 pts)
Answer:
8th Grade
7th Grade
Totals
Music
48
World Language
108
Totals
124
318

Answers

The students in 8th class who choose music are 16 in number,

The students in 7th class who choose music are 32 in number

The total who choose world language is 270

The totals who are in 7th grade be 194

The number of students who choose world language in 7th class are 162

Let x be the students in 8th class who choose music

x+108=124

x=124-108

x=16 students

Now  y be the students in 7th class who choose music

16+y=48

y=48-16

y=32

Let z be the total who choose world language

48+z=318

z=318-48

z=270

Now the totals who are in 7th grade be A

124+A=318

A=318-124

A=194

B be the number of students who choose world language in 7th class

Now 32+B=194

B=194-32

B=162

To learn more on Equation:

https://brainly.com/question/10413253

#SPJ1

Male 16 15 2 33
Female 14 11 18 43
Total 30 26 20 76
If one student is chosen at random,

Find the probability that the student was female:

Find the probability that the student was female AND got a "C":

Find the probability that the student was female OR got a "C":

If one student is chosen at random, find the probability that the student was female GIVEN they got a 'C':

Answers

a) The probability that the student was female is 43/76

b) The probability that the student was female AND got a "C" is 9/38

c) The probability that the student was female OR got a "C" is 45/76

We have,

             A   B   C  

Male     16  15  2  33

Female 14  11  18  43

Total     30 26 20 76

a) The probability that the student was female

= 43/76

b) The probability that the student was female AND got a "C"

= 18/76

= 9/38

c) The probability that the student was female OR got a "C"

= 43 /76 + 20/76 - 18/76

= 45/76

Learn more about Probability here:

https://brainly.com/question/30034780

#SPJ1

Question 2
Plot the coordinates on the number line provided and then find the coordinate of the indicated point
on a number line that partitions the segment into the given ratio.
(The numbers are not aligned properly to the marks, so redraw the number line.)
A is at -2 and B is at 14. Find the point, T, so that T is three-fourths of the distance from A to B.
-10
5
10

Answers

Note that  in the line graph presented, 3/4 the distance from A to B is 12, thus, t = 12.

what is the explanation about this?

Given:

A (- 2 , 0) and B(14, 0)

Using distance formula,      we can determine the distance between coordinates given:

d    = √ ( (x2 - x1)² + ( y2 - y1) ²)

In this case,  x1 = -2  y1 = 0,

x2 = 14, and y2 =  0.

Substituting these     values into the formula, we get:

d = √(  (14 -   (-2 ) ) ² + (0 - 0) ²)

= √((16)² + (0)² )

= √(256)

= 16

The distanc between the points (-2,0) and (14,0) is 16   units.

Recall that we need to find the position of t which    is 3/4 the distance between A and B.

That is:

3/4 x 16

t = 12.


Learn more about line graphs:
https://brainly.com/question/13298277
#SPJ1

Tomas used 3 [tex]\frac{1}{2}[/tex] of flour and now has 1 and two-thirds cups left. Which equation can he use to find f, the number of cups of flour he had to begin with?

f + 3 and one-third = 1 and two-thirds
f minus 3 and one-third = 1 and two-thirds
3 and one-third f = 1 and two-thirds
f divided by 3 and one-third = 1 and two-thirds

Answers

Answer:

B

Step-by-step explanation:

Answer:

I think It's b or a

Step-by-step explanation:

did this like 4 weeks ago.

Select the correct answer. What is the solution to 4|x − 3| + 1 = 1? A. x = 3 B. x = 3 or x = 4 C. x = 3 or x = 6 D. No solutions exist.

Answers

Answer:

a  

Step-by-step explanation:

We can start solving the given equation by isolating the absolute value term:

4|x - 3| = 0

Since the absolute value of any number is always non-negative, the only way for the left-hand side of the equation to be zero is if the absolute value term is zero. This means:

|x - 3| = 0

And the only solution to this equation is:

x - 3 = 0

x = 3

Therefore, the correct answer is A. x = 3.


A rectangular fish tank 35 cm long, 30 cm wide and 25 cm high is filled with water to a depth of 20 cm.
Find the volume of water in the fish tank.

Give your answer in liters.

Answers

Answer:

First, we need to calculate the volume of the rectangular fish tank. The volume of a rectangular solid is given by the formula:

volume = length x width x height

Plugging in the values we have, we get:

volume = 35 cm x 30 cm x 25 cm = 26,250 cubic cm

Next, we need to find the volume of water in the fish tank. The volume of water is equal to the volume of the space the water occupies, which is a rectangular solid with length 35 cm, width 30 cm, and height 20 cm. The volume of this rectangular solid is:

volume = length x width x height = 35 cm x 30 cm x 20 cm = 21,000 cubic cm

To convert cubic centimeters (cm^3) to liters (L), we need to divide by 1000:

volume = 21,000 cm^3 ÷ 1000 = 21 L

Therefore, the volume of water in the fish tank is 21 liters.

Step-by-step explanation:

PLEASE HELP (WILL GIVE BRAINLIEST

Answers

The exact volume of the soup can is 18.375 π cubic inches. So, correct option is C.

The volume of a cylinder can be found by multiplying the area of the base (circle) by the height of the cylinder. In this case, the diameter of the base is 3.5 inches, which means the radius is 1.75 inches (since radius = diameter/2).

So the area of the base can be calculated as:

π × (1.75 in)² = π × 3.0625 in²

The height of the cylinder is given as 6 inches.

Therefore, the exact volume of the soup can would be:

π × (1.75 in)² × 6 in = π × 3.0625 in² × 6 in = 18.375 π cubic inches (rounded to two decimal places)

So, correct option is C.

To learn more about volume click on,

https://brainly.com/question/9774275

#SPJ1

In this circle, mQR = 72°.
What is m/QPR?
A. 18°
B. 24°
C 36°
D. 72°

Please show work or give an explanation please

Answers

Answer:

< QPR = 36

Step-by-step explanation:

Inscribed Angle = 1/2 Intercepted Arc

< QPR = 1/2 ( 72)

< QPR = 36

72+z rewrite the expression as a produc 72

Answers

Rewriting the expression based on the information will be 72(1 + z/72).

How to explain the expression

In order to transform 72 + z into a product based on 72, we can apply the distributive property of multiplication over addition. Essentially, this property maintains that:

a x (b + c) = a x b + a x c

By extension, we can create the following equation:

72 + z = 72 + 1 x z

In order to produce an equivalent expression as a multiple of 72 and another factor, we would then need to use the distributive rule once again:

72 + 1 x z = 72 x 1 + 72 x z/72

Simplification leads us to arrive at:

72 + z = 72(1 + z/72)

Our end result is expressed as a synthesis between 72 and the expression (1 + z/72), namely:

72(1 + z/72).

Learn more about expressions on

https://brainly.com/question/1859113

#SPJ1

Marques is going to invest in an account paying an interest rate of 2% compounded
daily. How much would Marques need to invest, to the nearest dollar, for the value of
the account to reach $28,000 in 6 years?

Answers

We can use the formula for compound interest to solve this problem: A = P * (1 + r/n)^(n*t), where A is the final amount, P is the principal amount, r is the annual interest rate, n is the number of times interest is compounded per year, and t is the time in years.

In this case, we have:

A = $28,000
r = 2% = 0.02
n = 365 (since interest is compounded daily)
t = 6

We want to solve for P. Substituting the given values into the formula, we get:

$28,000 = P * (1 + 0.02/365)^(365*6)

Dividing both sides by (1 + 0.02/365)^(365*6), we get:

P = $28,000 / (1 + 0.02/365)^(365*6) = $22,406.57

Therefore, Marques would need to invest $22,407 (rounded to the nearest dollar) for the value of the account to reach $28,000 in 6 years.

The data set below gives the number of confirmed cases of malaria in Ethiopia.
Find the exponential model that best fits the data set and use the equation to estimate the number of malaria cases in 2017.



Year 2001 2002 2005 2007 2009 2010 2012

Malaria 392 428 539 451 1036 1158 1693

cases

In thousands

Answers

The number of malaria cases in 2017 is 2700.

Let the dependent variable, y is malaria cases in thousands and the independent variable, x is the year.

Here, x is the number of years starting in the year 2000.

The required model is obtained as [tex]y = 299e^{0.1295x}[/tex]

Now, the predicted number of malaria cases in the year 2017, that is, x = 17 years can be computed as,

[tex]y = 299e^{0.1295(17)}\\\\= 299(9.020\\\\= 2700[/tex]

Hence, the number of malaria cases in 2017 is 2700.

Learn more about exponential model click;

https://brainly.com/question/28596571

#SPJ1

What function is represented in the table?
Select one:
y = 3(2^x)
y = 2(3^x)
y = 2(.5^x)
y = 3^x

Answers

The exponential function represented in the table is given as follows:

y = 3^x.

How to define an exponential function?

An exponential function has the definition presented as follows:

y = ab^x.

In which the parameters are given as follows:

a is the value of y when x = 0.b is the rate of change.

From the table, we have that when x=  0, y = 1, hence the parameter a is given as follows:

a = 1.

When x is increased by one, y is multiplied by three, hence the parameter b is given as follows:

b = 3.

Hence the function is:

y = 3^x.

More can be learned about exponential functions at brainly.com/question/2456547

#SPJ1

Help please!! I need help!!! I will mark you brainlest!​

Answers

Answer:

2(4.5) + (1/2)(4)(2) = 9 + 4 = 13 square feet

13 square feet it is

Michael checked several websites and stores around town for the television he wanted to purchase. He saw eight different prices: $273 $267 $276 $297 $264 $294 $264 $269 What is the mean, median, and mode of this data set?

Answers

The mean is 270.75 and the median is 271. While there is no single mode, then we conclude that our mode is 264 and 294 respectively.

Understanding how to calculate mean, median, mode

By definition:

Mean is the sum of all the numbers in the set divided by the total number of numbers.

Median is the middle value in the set when the numbers are arranged in order.

Mode is the number that appears most frequently in the set.

If we re-arrange the data given in ascending order, we have:

264 264 267 269 273 276 294 297

To calculate Mean:

Add up all the numbers and divide by the total number of numbers:

Mean = (264 + 264 + 267 + 269 + 273 + 276 + 294 + 297) / 8

Mean = 270.75

To calculate Median:

We need to find the middle number in the set. Since there are 8 numbers in the set, the median is the average of the 4th and 5th numbers:

Median = (269 + 273) / 2

Median = 271

To calculate Mode:

We need to determine which number appears most frequently in the set. In this case, there are two numbers that appear twice (264 and 294), and the rest of the numbers appear only once. Therefore, there is no single mode for this data set.

Mode = 264 and 294

Learn more about median here:

https://brainly.com/question/26177250

#SPJ1

what is the volume of a hemisphere with a diameter of 8ft (rounded to the nearest tenth of a cubic foot)

Answers

well, if the hemisphere has a diameter of 8, that means its radius is half that or 4, hmm let's find the volume of a Sphere with such a radius, then let's halve it.

[tex]\textit{volume of a sphere}\\\\ V=\cfrac{4\pi r^3}{3}~~ \begin{cases} r=radius\\[-0.5em] \hrulefill\\ r=4 \end{cases}\implies V=\cfrac{4\pi (4)^3}{3} \\\\\\ \stackrel{\textit{now let's get half of that for the hemisphere}}{V=\cfrac{1}{2}\cdot \cfrac{4\pi (4)^3}{3}\implies V=\cfrac{128\pi }{3}}\implies \boxed{V\approx 134.0}[/tex]

if i have 7 shirts and 2 pants how many outfits can i make​

Answers

Answer:14

Step-by-step explanation:

2 pants can only go with the different shirts once. So it should be 14

Which compares the end behavior of the functions f and g?
f(x) = −17x − 9 g(x) = − 78
7
8
x + 20
A. For function f, as x → ∞
x




, f(x) → ∞
f
(
x
)




. Likewise, for function g, as x → ∞
x




, g(x) → ∞
g
(
x
)




.
B. For function f, as x → ∞

x




, f(x) → −∞
f
(
x
)





. Likewise, for function g, as x → ∞
x




, g(x) → −∞
g
(
x
)





.
C. For function f, as x → ∞
x




, f(x) → −∞
f
(
x
)



-

. However, for function g, as x → ∞
x




, g(x) → ∞
g
(
x
)




.
D. For function f, as x → ∞

x




, f(x) → ∞

f
(
x
)




. However, for function g, as x → ∞
x




, g(x) → −∞
g
(
x
)





.

Answers

Answer:

The correct option that compares the end behavior of the functions f and g is D.

For function f, as x → ∞, f(x) → -∞, which means that the function approaches negative infinity as x approaches infinity. This is because the leading term of the function is -17x, which approaches negative infinity as x approaches infinity.

For function g, as x → ∞, g(x) → -∞, which means that the function also approaches negative infinity as x approaches infinity. This is because the leading term of the function is -78/87x, which approaches negative infinity as x approaches infinity.

Therefore, both functions have the same end behavior, which is approaching negative infinity as x approaches infinity.

Which number line shows a graph of the inequality x < 6

Answers

The number line that shows the inequality x < 6 is given by the following option:

Option D.

What are the inequality symbols?

The four inequality symbols, along with their meaning on the number line and the coordinate plane, are presented as follows:

> x: the amount is greater than x -> the number is to the right of x with an open dot at the number line. -> points above the dashed horizontal line y = x on the coordinate plane.< x: the amount is less than x. -> the number is to the left of x with an open dot at the number line. -> points below the dashed horizontal line y = x on the coordinate plane.≥ x: the amount is at least x. -> the number is to the right of x with a closed dot at the number line. -> points above the solid vertical line y = x on the coordinate plane.≤ the amount is at most x. -> the number is to the left of x with a closed dot at the number line. -> points above the dashed vertical line y = x on the coordinate plane.

The inequality for this problem is given as follows:

x < 6.

Which is composed by the numbers to the left of x = 6, with an open circle, due to the open interval.

More can be learned about inequalities at brainly.com/question/25275758

#SPJ1

PLEASEEEE HELP MEMEMEMEM PLESSEEE I WILL GIVE U BRAINLIST QUICK

Answers

The numeric value of the function graphed is given as follows:

f(-8) = 6.

How to obtain the numeric value of the function?

The expression for the numeric value of the function in this problem is given as follows:

f(-8).

This means that the input is given as follows:

x = -8.

Passing a vertical line through x = -8, the value of y in which the vertical line crosses the graph is given as follows:

y = 6.

Hence the numeric value is given as follows:

f(-8) = 6.

Learn more about the numeric values of a function at brainly.com/question/28367050

#SPJ1

Which function is a translation one unit right of the function f(x) = log x?

h(x) = log x + 1
g(x) = log x-1
j(x) = log(x + 1)
k(x) = log(x - 1)

Answers

Answer:

The function that is a translation one unit right of the function f(x) = log x is k(x) = log(x - 1).

When we shift a function f(x) one unit right, we replace x with (x - a) where "a" is the amount of the shift. In this case, "a" is equal to 1. Therefore, the function becomes:

f(x - 1) = log(x - 1)

This means that the function k(x) = log(x - 1) is obtained by shifting the function f(x) one unit to the right.

How many times can u write your name in 12 minutes using variables Based on your recorded values, write an equation that represents the relationship between the time you spent doing the activity and the amount of the activity you completed. Make sure to define your variables

:D

Answers

The following equation denotes the correlation between the duration and output of an activity: A = kT

How to explain the equation

It should be noted that A represents the amount of production, T is the period of working on the task, and k consists of a constant ratio.

Therefore, it can be conclusively stated that the culminating yield is in direct correlation to the amount of effort contributed, as denoted by the factor of proportionality k, thus producing a linear relationship when displayed upon a graph.

Learn more about equations on

https://brainly.com/question/2972832

#SPJ1

PLEASE HELP ILL DO SNYTHING PLSPLSPLS

5. Beth is 63 years old. She loves to bike. She decides to look at other clubs to join. Why do you think she didn’t want to join this club? Explain your answer using the stem-and-leaf plot above. (1 point)

Answers

Beth could not join the club because the highest age which is allowed here is only 55 years

Since, A statistical expression obtained from a list of data that refers an abnormal gap from other values.

The statistical rules that instruct us to divides the data or observation values into four parts.

After analyzing the stem leaf diagram, we noticed the youngest age allowed to join the club is 10 years and the club allow highest 55 years old to join.

the plot also defines that the number of members is declined with the age. There is only person in the club who is 55 years. There are few members with the age over 40 years.

hence, Beth did not join the club because her age was above the highest age that allowed in this club.

to know more about Stem Leaf diagram visit:

brainly.com/question/29479023

#SPJ1

PLEASE HELP ( WILL GIVE BRAINLIEST)

Answers

The amount of money Laurie would pay to have the pond cleaned is equal to $3,161.85.

How to calculate the volume of a hemisphere?

In Mathematics and Geometry, the volume of a hemisphere can be calculated by using the following mathematical equation (formula):

Volume of a hemisphere = 2/3 × πr³

Where:

r represents the radius.

Note: Radius = diameter/2 = 13/2 = 6.5 feet.

By substituting the given parameters into the formula for the volume of a hemisphere, we have the following;

Volume of a hemisphere = 2/3 × 3.14 × (6.5)³

Volume of a hemisphere = 574.881 ft³

Cost = 5.50 × 574.881 ft³

Cost = $3,161.85

Read more on volume of a sphere here: brainly.com/question/20394302

#SPJ1

Find the values of x and y

Answers

Answer:

x = 8.4

y = 23.6

Step-by-step explanation:

tan31 = x/14

x = tan31(14) = 8.412

y² = (8.412)² + (22)² = 554.7626

y = √554.7626 = 23.55

Find the volume of rectangular prism. (You should have at least 2 steps shows in your work!)(show all decimal places or a fraction)

Answers

The volume of this rectangular prism include the following: 84.375 cubic meters.

How to calculate the volume of a rectangular prism?

In Mathematics and Geometry, the volume of a rectangular prism can be calculated by using the following formula:

Volume of a rectangular prism = L × W × H

Where:

L represents the length of a rectangular prism.W represents the width of a rectangular prism.H represents the height of a rectangular prism.

By substituting the given dimensions (parameters) into the formula for the volume of a rectangular prism, we have the following;

Volume of rectangular prism = 5 × 4 1/2 ×  3 3/4

Volume of rectangular prism = 5 × 4.5 ×  3.75

Volume of rectangular prism = 84.375 cubic meters.

Read more on volume of prism here: brainly.com/question/21012007

#SPJ1

Hurry up please time limit

State the domain and range and tell if the graph is a function yes or no?
What’s the domain and range ?

Answers

Answer:No, X=-3, yly =-4,0,4

Step-by-step explanation:

Do the following using Matlab: Generate 10 numbers between 1 and 6, calculate the mean. Generate 100 numbers between 1 and 6. calculate the mean. Generate 1000 numbers between 1 and 6, calculate the mean. There is a command in Matlab to generate numbers. Do not include the numbers when you submit the project. The project should show the commands used and the results only The idea is to roll a die using Matlab commands, instead of doing it manually. Roll a die 10 times and record the outcome and then find the average of the outcomes. Then compare the means for the cases of 10, 100 & 1000. On page 2 of lecture 10, we found that the mean of rolling a die to be 3.5. Therefore, your answer will be close to 3.5 every time you repeat the experiment for larger numbers.

Answers

We can see that the means we got from Matlab are close to 3.5 for larger numbers of rolls.

We have,

To generate numbers between 1 and 6 in Matlab, we can use the "randi" command.

To generate 10 numbers, we can use:
```matlab
numbers10 = randi([1, 6], [1, 10]);
mean10 = mean(numbers10);
disp(mean10);
```

To generate 100 numbers, we can use:
```matlab
numbers100 = randi([1, 6], [1, 100]);
mean100 = mean(numbers100);
disp(mean100);
```

To generate 1000 numbers, we can use:
```matlab
numbers1000 = randi([1, 6], [1, 1000]);
mean1000 = mean(numbers1000);
disp(mean1000);
```

After running these commands, we will get the means for generating 10, 100, and 1000 numbers between 1 and 6.

As mentioned in the question, the mean of rolling a die is 3.5.

Therefore,

We can see that the means we got from Matlab are close to 3.5 for larger numbers of rolls.

Learn more about math lab here:

https://brainly.com/question/10772022

#SPJ11

Find the missing angle

Answers

Answer:

I Think 62°

Step-by-step explanation:

I think it’s the answer is 62

I need help with these

Answers


19. __C__ Segment QC is the radius (1/2 diameter)

20. __K__ Arc BDE is major because three points are listed

21. __E__ Segment AD is the diameter (length cutting the circle in half)

22. __L__ Arc ABD has two endpoints on the circle’s diameter

23. __H__ Angle AQB has an apex at the circle’s center, Q

24. __A__ Point Q is the circle’s center

25. __D__ Segment ED is a chord as it has endpoints on the circular arc

26. __B__ Point C is perpendicular to the radius (QC) where it meets the circle.

27. __F__ Segment AB is a secant because it contains a chord and intersects the circle at 2 points.

28. __J__ Arc EA is minor between two points

29. __G__ Segment CG is the tangent as it intersects the circle at one point, C.


Hope this helps!
Other Questions
22. The _____ Act protects the retirement income of employees and retirees. and include it in the show your work file attached to question Given the homogeneous system of linear equations, work items a, b, cand type the final answers in the answer box, Write legibly to show all the steps to the final answers x-2y+32-0 -3x+6y-92=0 a (7.5 pts.) Find a basis for its solution space (nullspace of the coefficient matrix) b- (5 pts) What is the dimension of the solution space? (nullity of the coefficient matrix) c-(7.5 pts.) Find a basis for row space of the coefficient matrix What are characteristics of a moist, unstable air mass?A.Turbulence and showery precipitation. B.Poor visibility and smooth air.C.Haze and smoke. The variable b varies directly as the square root of c. If b = 100 when c = 4, which equation can be used to find other combinations of b and c?a: b = 200cb: b = 50cc: b = 25cd: bc = 50 What types of accounts appear in the unadjusted trial balance? What are the purposes of this trial balance? Rewrite the following compound sentences as complex sentences: 1 The thief saw the police man, so he raw away. Assignment I 1. Prepare a questionare on Expenditures and Consumptions behaviours and attitudes of polytechnic Students (Case Study of TOPS) Nothing more than. He was a ____ child when he wrote the book select all that apply under us gaap, a contingent liability should . (enter one word per blank.) multiple select question. not be reported if the loss is remote and unable to be estimated be reported on the balance sheet if the loss will probably occur and can be reasonably estimated be in the notes to the financial statements if the loss may possibly occur and can be reasonably estimated be reported on the balance sheet if the loss may possibly occur and can be reasonably estimated Question 1 Solve the following differential equations leaving your answer in the form x a. dx/dy = 5x/y ii) = dx/dy= x^4 Physical elements needed to promote quality in educational context If minimum observation is 47.6 and maximum observation is 128.4, number of classes is 6, then the third class and the midpoint of the fourth class respectively are:a.[74.5 87.9] and 94.75 b.[74.6 88.0] and 94.8 c.[74.5 88.2] and 94.7d.[74.7 88.1] and 94.75 e.[74.6 88.1] and 94.8 A dt controller is designed for a system to achieve closed loop bandwidth 10hz. Estimate a reasonable sampling time to discretize such a controller using the phase margin rule and the rise time rule Signature-based detection describes a form of intrusion detection system/intrusion prevention system (IDS/ IPS) based on a defined normal. true or false Ahrends Corporation makes 70,000 units per year of a part it uses in the products it manufactures. The unit product cost of this part is computed as follows: Direct materials $ 17. 80 Direct labor 19. 00 Variable manufacturing overhead 1. 00 Fixed manufacturing overhead 17. 10 Unit product cost $ 54. 90 An outside supplier has offered to sell the company all of these parts it needs for $48. 50 a unit. If the company accepts this offer, the facilities now being used to make the part could be used to make more units of a product that is in high demand. The additional contribution margin on this other product would be $273,000 per year. If the part were purchased from the outside supplier, all of the direct labor cost of the part would be avoided. However, $8. 20 of the fixed manufacturing overhead cost being applied to the part would continue even if the part were purchased from the outside supplier. This fixed manufacturing overhead cost would be applied to the company's remaining products. How much of the unit product cost of $54. 90 is relevant in the decision of whether to make or buy the part? Paula Boothe, president of the Blue Corporation, has mandated a minimum 8% return on investment for any project undertaken by the company. Given the company's decentralization, Paula leaves all investment decisions to the divisional managers as long as they anticipate a minimum rate of return of at least 10%. The Energy Drinks division, under the direction of manager Martin Koch, has achieved a 16% return on investment for the past three years. This year is not expected to be different from the past three. Koch has just received a proposal to invest $1,916,000 in a new line of energy drinks that is expected to generate $230,700 in operating income. (a) Calculate the residual income for the proposed new line of energy drinks. Residual income $ e Textbook and Media Save for Later Attempts: 0 of 3 used Submit Answer A sunscreen manufacturer tests a new water-resistant sunscreen by finding 120 volunteers at a community pool. The subjects are randomly assigned to one of two groups by drawing either a 1 or 2 marked on slips of equal-sized paper from a bag. Half of the subjects use an old formulation, and half use the new formulation over an 8-hour time frame on the same sunny day. Test administrators apply a measured amount of sunscreen to each subject at the beginning of the day and reapply it at regular intervals throughout the day. Test subjects are asked to remain in unshaded areas as much as possible during the test. The levels of sunburn are compared at the end of the day. What are the three methods to transfer depth dimensions?dividers, scale, & miter linedividers, compass, & miter linescale, compass, & dividersscale, compass, & miter linedividers, miter line, & centerline method Math Algebra Help needed HELP DUE TODAY i got a another one in my profile please this is 1 assignment plss give me right answer